Difference between revisions of "1999 AHSME Problems"

(create)
 
(Problem 2: better formatting)
 
(17 intermediate revisions by 12 users not shown)
Line 1: Line 1:
 +
{{AHSME Problems|year=1999}}
 
== Problem 1 ==
 
== Problem 1 ==
 
<math>1 - 2 + 3 -4 + \cdots - 98 + 99 = </math>
 
<math>1 - 2 + 3 -4 + \cdots - 98 + 99 = </math>
Line 9: Line 10:
 
Which of the following statements is false?
 
Which of the following statements is false?
  
<math> \mathrm{(A) \ All\ equilateral\ triangles\ are\ congruent\ to\ each\ other.}</math>
+
<math>\text{(A) All equilateral triangles are congruent to each other.}</math>
<math>\mathrm{(B) All\ equilateral\ triangles\ are\ convex.}</math>
+
 
<math>\mathrm{(C) All\ equilateral\ triangles\ are\ equianguilar.}</math>
+
<math>\text{(B) All equilateral triangles are convex.}</math>
<math>\mathrm{(D) All\ equilateral\ triangles\ are\ regular\ polygons.}</math>
+
 
<math>\mathrm{(E) All\ equilateral\ triangles\ are\ similar\ to\ each\ other.} </math>
+
<math>\text{(C) All equilateral triangles are equiangular.}</math>
 +
 
 +
<math>\text{(D) All equilateral triangles are regular polygons.}</math>
 +
 
 +
<math>\text{(E) All equilateral triangles are similar to each other.}</math>
  
 
[[1999 AHSME Problems/Problem 2|Solution]]
 
[[1999 AHSME Problems/Problem 2|Solution]]
  
 
== Problem 3 ==
 
== Problem 3 ==
 +
The number halfway between <math>1/8</math> and <math>1/10</math> is
  
<math> \mathrm{(A) \  } \qquad \mathrm{(B) \  } \qquad \mathrm{(C) \  } \qquad \mathrm{(D) \  } \qquad \mathrm{(E) \  } </math>
+
<math> \mathrm{(A) \  } \frac 1{80} \qquad \mathrm{(B) \  } \frac 1{40} \qquad \mathrm{(C) \  } \frac 1{18} \qquad \mathrm{(D) \  } \frac 1{9} \qquad \mathrm{(E) \  } \frac 9{80} </math>
  
 
[[1999 AHSME Problems/Problem 3|Solution]]
 
[[1999 AHSME Problems/Problem 3|Solution]]
  
 
== Problem 4 ==
 
== Problem 4 ==
 +
Find the sum of all prime numbers between <math>1</math> and <math>100</math> that are simultaneously <math>1</math> greater than a multiple of <math>4</math> and <math>1</math> less than a multiple of <math>5</math>.
  
<math> \mathrm{(A) \ } \qquad \mathrm{(B) \ } \qquad \mathrm{(C) \ } \qquad \mathrm{(D) \ } \qquad \mathrm{(E) \ } </math>
+
<math> \mathrm{(A) \ } 118 \qquad \mathrm{(B) \ }137 \qquad \mathrm{(C) \ } 158 \qquad \mathrm{(D) \ } 187 \qquad \mathrm{(E) \ } 245</math>
  
 
[[1999 AHSME Problems/Problem 4|Solution]]
 
[[1999 AHSME Problems/Problem 4|Solution]]
  
 
== Problem 5 ==
 
== Problem 5 ==
 +
The marked price of a book was <math>30 \%</math> less than the suggested retail price. Alice purchased the book for half the marked price at a Fiftieth Anniversary sale. What percent of the suggested retail price did Alice pay?
  
<math> \mathrm{(A) \ } \qquad \mathrm{(B) \ } \qquad \mathrm{(C) \ } \qquad \mathrm{(D) \ } \qquad \mathrm{(E) \ } </math>
+
<math> \mathrm{(A) \ }25 \% \qquad \mathrm{(B) \ }30 \% \qquad \mathrm{(C) \ }35 \% \qquad \mathrm{(D) \ }60 \% \qquad \mathrm{(E) \ }65 \% </math>
  
 
[[1999 AHSME Problems/Problem 5|Solution]]
 
[[1999 AHSME Problems/Problem 5|Solution]]
  
 
== Problem 6 ==
 
== Problem 6 ==
 +
What is the sum of the digits of the decimal form of the product <math>2^{1999} \cdot 5^{2001}</math>?
  
<math> \mathrm{(A) \ } \qquad \mathrm{(B) \ } \qquad \mathrm{(C) \ } \qquad \mathrm{(D) \ } \qquad \mathrm{(E) \ } </math>
+
<math> \mathrm{(A) \ }2 \qquad \mathrm{(B) \ }4 \qquad \mathrm{(C) \ }5 \qquad \mathrm{(D) \ }7 \qquad \mathrm{(E) \ }10 </math>
  
 
[[1999 AHSME Problems/Problem 6|Solution]]
 
[[1999 AHSME Problems/Problem 6|Solution]]
  
 
== Problem 7 ==
 
== Problem 7 ==
 +
What is the largest number of acute angles that a convex hexagon can have?
  
<math> \mathrm{(A) \ } \qquad \mathrm{(B) \ } \qquad \mathrm{(C) \ } \qquad \mathrm{(D) \ } \qquad \mathrm{(E) \ } </math>
+
<math> \mathrm{(A) \ }2 \qquad \mathrm{(B) \ }3 \qquad \mathrm{(C) \ }4 \qquad \mathrm{(D) \ }5 \qquad \mathrm{(E) \ }6 </math>
  
 
[[1999 AHSME Problems/Problem 7|Solution]]
 
[[1999 AHSME Problems/Problem 7|Solution]]
  
 
== Problem 8 ==
 
== Problem 8 ==
 +
At the end of 1994 Walter was half as old as his grandmother. The sum of the years in which they were born is 3838. How old will Walter be at the end of 1999?
  
<math> \mathrm{(A) \ } \qquad \mathrm{(B) \ } \qquad \mathrm{(C) \ } \qquad \mathrm{(D) \ } \qquad \mathrm{(E) \ } </math>
+
<math> \mathrm{(A) \ } 48 \qquad \mathrm{(B) \ }49 \qquad \mathrm{(C) \ }53 \qquad \mathrm{(D) \ }55 \qquad \mathrm{(E) \ } 101</math>
  
 
[[1999 AHSME Problems/Problem 8|Solution]]
 
[[1999 AHSME Problems/Problem 8|Solution]]
  
 
== Problem 9 ==
 
== Problem 9 ==
 +
Before Ashley started a three-hour drive, her car's odometer reading was 29792, a palindrome. (A palindrome is a number that reads the same way from left to right as it does from right to left). At her destination, the odometer reading was another palindrome. If Ashley never exceeded the speed limit of 75 miles per hour, which of the following was her greatest possible average speed?
  
<math> \mathrm{(A) \ } \qquad \mathrm{(B) \ } \qquad \mathrm{(C) \ } \qquad \mathrm{(D) \ } \qquad \mathrm{(E) \ } </math>
+
<math> \mathrm{(A) \ } 33\frac 13 \qquad \mathrm{(B) \ }53\frac 13 \qquad \mathrm{(C) \ }66\frac 23 \qquad \mathrm{(D) \ }70\frac 13 \qquad \mathrm{(E) \ } 74\frac 13</math>
  
 
[[1999 AHSME Problems/Problem 9|Solution]]
 
[[1999 AHSME Problems/Problem 9|Solution]]
  
 
== Problem 10 ==
 
== Problem 10 ==
 +
A sealed envelope contains a card with a single digit on it. Three of the following statements are true, and the other is false.
 +
 +
I. The digit is 1.
 +
 +
II. the digit is not 2.
 +
 +
III. The digit is 3.
 +
 +
IV. The digit is not 4.
 +
 +
Which one of the following must necessarily be correct?
  
<math> \mathrm{(A) \ } \qquad \mathrm{(B) \ } \qquad \mathrm{(C) \ } \qquad \mathrm{(D) \ } \qquad \mathrm{(E) \ } </math>
+
<math> \mathrm{(A) \ I\ is\ true} \qquad \mathrm{(B) \ I\ is\ false} \qquad \mathrm{(C) \ II\ is\ true} \qquad \mathrm{(D) \ III\ is\ true} \qquad \mathrm{(E) \ IV\ is\ false} </math>
  
 
[[1999 AHSME Problems/Problem 10|Solution]]
 
[[1999 AHSME Problems/Problem 10|Solution]]
  
 
== Problem 11 ==
 
== Problem 11 ==
 +
The student lockers at Olympic High are numbered consecutively beginning with locker number <math>1</math>. The plastic digits used to number the lockers cost two cents apiece. Thus, it costs two cents to label locker number <math>9</math> and four cents to label locker number <math>10</math>. If it costs <math>137.94</math> to label all the lockers, how many lockers are there at the school?
  
<math> \mathrm{(A) \ } \qquad \mathrm{(B) \ } \qquad \mathrm{(C) \ } \qquad \mathrm{(D) \ } \qquad \mathrm{(E) \ } </math>
+
<math> \mathrm{(A) \ }2001 \qquad \mathrm{(B) \ }2010 \qquad \mathrm{(C) \ }2100 \qquad \mathrm{(D) \ }2726 \qquad \mathrm{(E) \ }6897 </math>
  
 
[[1999 AHSME Problems/Problem 11|Solution]]
 
[[1999 AHSME Problems/Problem 11|Solution]]
  
 
== Problem 12 ==
 
== Problem 12 ==
 +
What is the maximum number of points of intersection of the graphs of two different fourth degree polynomial functions <math>y = p(x)</math> and <math>y = q(x)</math>, each with leading coefficient <math>1</math>?
  
<math> \mathrm{(A) \ } \qquad \mathrm{(B) \ } \qquad \mathrm{(C) \ } \qquad \mathrm{(D) \ } \qquad \mathrm{(E) \ } </math>
+
<math>\textrm{(A)} \ 1 \qquad \textrm{(B)} \ 2 \qquad \textrm{(C)} \ 3 \qquad \textrm{(D)} \ 4 \qquad \textrm{(E)} \ 8</math>
  
 
[[1999 AHSME Problems/Problem 12|Solution]]
 
[[1999 AHSME Problems/Problem 12|Solution]]
  
 
== Problem 13 ==
 
== Problem 13 ==
 +
Define a sequence of real numbers <math>a_1, a_2, a_3, \ldots</math> by <math>a_1 = 1</math> and <math>a_{n+1}^3 = 99a_n^3</math> for all <math>n \ge 1</math>. Then <math>a_{100}</math> equals
  
<math> \mathrm{(A) \ } \qquad \mathrm{(B) \ } \qquad \mathrm{(C) \ } \qquad \mathrm{(D) \ } \qquad \mathrm{(E) \ } </math>
+
<math> \mathrm{(A) \ } 33^{33} \qquad \mathrm{(B) \ } 33^{99} \qquad \mathrm{(C) \ } 99^{33} \qquad \mathrm{(D) \ }99^{99} \qquad \mathrm{(E) \ none\ of\ the\ above} </math>
  
 
[[1999 AHSME Problems/Problem 13|Solution]]
 
[[1999 AHSME Problems/Problem 13|Solution]]
  
 
== Problem 14 ==
 
== Problem 14 ==
 +
Four girls - Mary, Aline, Tina, and Hana - sang songs in a concert as trios, with one girl sitting out each time. Hanna sang 7 songs, which was more than any other girl, and Mary sang 4 songs, which was fewer than any other girl. How many songs did these trios sing?
  
<math> \mathrm{(A) \ } \qquad \mathrm{(B) \ } \qquad \mathrm{(C) \ } \qquad \mathrm{(D) \ } \qquad \mathrm{(E) \ } </math>
+
<math> \mathrm{(A) \ 7 } \qquad \mathrm{(B) \ 8 } \qquad \mathrm{(C) \ 9 } \qquad \mathrm{(D) \ 10 } \qquad \mathrm{(E) \ 11 } </math>
  
 
[[1999 AHSME Problems/Problem 14|Solution]]
 
[[1999 AHSME Problems/Problem 14|Solution]]
  
 
== Problem 15 ==
 
== Problem 15 ==
 +
Let <math>x</math> be a real number such that <math>\sec x - \tan x = 2</math>. Then <math>\sec x + \tan x =</math>
  
<math> \mathrm{(A) \ } \qquad \mathrm{(B) \ } \qquad \mathrm{(C) \ } \qquad \mathrm{(D) \ } \qquad \mathrm{(E) \ } </math>
+
<math> \mathrm{(A) \ } 0.1 \qquad \mathrm{(B) \ } 0.2 \qquad \mathrm{(C) \ } 0.3 \qquad \mathrm{(D) \ } 0.4 \qquad \mathrm{(E) \ } 0.5</math>
  
 
[[1999 AHSME Problems/Problem 15|Solution]]
 
[[1999 AHSME Problems/Problem 15|Solution]]
  
 
== Problem 16 ==
 
== Problem 16 ==
 +
What is the radius of a circle inscribed in a rhombus with diagonals of length <math>10</math> and <math>24</math>?
  
<math> \mathrm{(A) \ } \qquad \mathrm{(B) \ } \qquad \mathrm{(C) \ } \qquad \mathrm{(D) \ } \qquad \mathrm{(E) \ } </math>
+
<math> \mathrm{(A) \ }4 \qquad \mathrm{(B) \ }\frac {58}{13} \qquad \mathrm{(C) \ }\frac{60}{13} \qquad \mathrm{(D) \ }5 \qquad \mathrm{(E) \ }6 </math>
  
 
[[1999 AHSME Problems/Problem 16|Solution]]
 
[[1999 AHSME Problems/Problem 16|Solution]]
  
 
== Problem 17 ==
 
== Problem 17 ==
 +
Let <math>P(x)</math> be a polynomial such that when <math>P(x)</math> is divided by <math>x-19</math>, the remainder is <math>99</math>, and when <math>P(x)</math> is divided by <math>x - 99</math>, the remainder is <math>19</math>. What is the remainder when <math>P(x)</math> is divided by <math>(x-19)(x-99)</math>?
  
<math> \mathrm{(A) \ } \qquad \mathrm{(B) \ } \qquad \mathrm{(C) \ } \qquad \mathrm{(D) \ } \qquad \mathrm{(E) \ } </math>
+
<math> \mathrm{(A) \ } -x + 80 \qquad \mathrm{(B) \ } x + 80 \qquad \mathrm{(C) \ } -x + 118 \qquad \mathrm{(D) \ } x + 118 \qquad \mathrm{(E) \ } 0</math>
  
 
[[1999 AHSME Problems/Problem 17|Solution]]
 
[[1999 AHSME Problems/Problem 17|Solution]]
  
 
== Problem 18 ==
 
== Problem 18 ==
 +
How many zeros does <math>f(x) = \cos(\log x)</math> have on the interval <math>0 < x < 1</math>?
  
<math> \mathrm{(A) \ } \qquad \mathrm{(B) \ } \qquad \mathrm{(C) \ } \qquad \mathrm{(D) \ } \qquad \mathrm{(E) \ } </math>
+
<math> \mathrm{(A) \ } 0 \qquad \mathrm{(B) \ } 1 \qquad \mathrm{(C) \ } 2 \qquad \mathrm{(D) \ } 10 \qquad \mathrm{(E) \ } \text{infinitely\ many}</math>
  
 
[[1999 AHSME Problems/Problem 18|Solution]]
 
[[1999 AHSME Problems/Problem 18|Solution]]
Line 115: Line 146:
 
== Problem 19 ==
 
== Problem 19 ==
  
<math> \mathrm{(A) \ } \qquad \mathrm{(B) \ } \qquad \mathrm{(C) \ } \qquad \mathrm{(D) \ } \qquad \mathrm{(E) \ } </math>
+
Consider all triangles <math>ABC</math> satisfying in the following conditions: <math>AB = AC</math>, <math>D</math> is a point on <math>\overline{AC}</math> for which <math>\overline{BD} \perp \overline{AC}</math>, <math>AC</math> and <math>CD</math> are integers, and <math>BD^{2} = 57</math>.  Among all such triangles, the smallest possible value of <math>AC</math> is
 +
 
 +
<asy>
 +
pair A,B,C,D;
 +
A=(5,12); B=origin; C=(10,0); D=(8.52071005917,3.55029585799);
 +
draw(A--B--C--cycle); draw(B--D);
 +
label("$A$",A,N); label("$B$",B,SW); label("$C$",C,SE); label("$D$",D,NE);
 +
</asy>
 +
 
 +
<math>\textrm{(A)} \ 9 \qquad \textrm{(B)} \ 10 \qquad \textrm{(C)} \ 11 \qquad \textrm{(D)} \ 12 \qquad \textrm{(E)} \ 13</math>  
  
 
[[1999 AHSME Problems/Problem 19|Solution]]
 
[[1999 AHSME Problems/Problem 19|Solution]]
  
 
== Problem 20 ==
 
== Problem 20 ==
 +
The sequence <math>a_{1},a_{2},a_{3},\ldots</math> satisfies <math>a_{1} = 19,a_{9} = 99</math>, and, for all <math>n\geq 3</math>, <math>a_{n}</math> is the arithmetic mean of the first <math>n - 1</math> terms.  Find <math>a_2</math>.
  
<math> \mathrm{(A) \ } \qquad \mathrm{(B) \ } \qquad \mathrm{(C) \ } \qquad \mathrm{(D) \ } \qquad \mathrm{(E) \ } </math>
+
<math>\textrm{(A)} \ 29 \qquad \textrm{(B)} \ 59 \qquad \textrm{(C)} \ 79 \qquad \textrm{(D)} \ 99 \qquad \textrm{(E)} \ 179</math>
  
 
[[1999 AHSME Problems/Problem 20|Solution]]
 
[[1999 AHSME Problems/Problem 20|Solution]]
  
 
== Problem 21 ==
 
== Problem 21 ==
 +
A circle is circumscribed about a triangle with sides <math>20,21,</math> and <math>29,</math> thus dividing the interior of the circle into four regions. Let <math>A,B,</math> and <math>C</math> be the areas of the non-triangular regions, with <math>C</math> be the largest. Then
  
<math> \mathrm{(A) \ } \qquad \mathrm{(B) \ } \qquad \mathrm{(C) \ } \qquad \mathrm{(D) \ } \qquad \mathrm{(E) \ } </math>
+
<math> \mathrm{(A) \ }A+B=C \qquad \mathrm{(B) \ }A+B+210=C \qquad \mathrm{(C) \ }A^2+B^2=C^2 \qquad \mathrm{(D) \ }20A+21B=29C \qquad \mathrm{(E) \ } \frac 1{A^2}+\frac 1{B^2}= \frac 1{C^2}</math>
  
 
[[1999 AHSME Problems/Problem 21|Solution]]
 
[[1999 AHSME Problems/Problem 21|Solution]]
  
 
== Problem 22 ==
 
== Problem 22 ==
 +
The graphs of <math>y = -|x-a| + b</math> and <math>y = |x-c| + d</math> intersect at points <math>(2,5)</math> and <math>(8,3)</math>. Find <math>a+c</math>.
  
<math> \mathrm{(A) \ } \qquad \mathrm{(B) \ } \qquad \mathrm{(C) \ } \qquad \mathrm{(D) \ } \qquad \mathrm{(E) \ } </math>
+
<math> \mathrm{(A) \ } 7 \qquad \mathrm{(B) \ } 8 \qquad \mathrm{(C) \ } 10 \qquad \mathrm{(D) \ } 13\qquad \mathrm{(E) \ } 18</math>
  
 
[[1999 AHSME Problems/Problem 22|Solution]]
 
[[1999 AHSME Problems/Problem 22|Solution]]
  
 
== Problem 23 ==
 
== Problem 23 ==
 +
The equiangular convex hexagon <math>ABCDEF</math> has <math>AB = 1, BC = 4, CD = 2,</math> and <math>DE = 4.</math> The area of the hexagon is
  
<math> \mathrm{(A) \ } \qquad \mathrm{(B) \ } \qquad \mathrm{(C) \ } \qquad \mathrm{(D) \ } \qquad \mathrm{(E) \ } </math>
+
<math> \mathrm{(A) \ } \frac {15}2\sqrt{3} \qquad \mathrm{(B) \ }9\sqrt{3} \qquad \mathrm{(C) \ }16 \qquad \mathrm{(D) \ }\frac{39}4\sqrt{3} \qquad \mathrm{(E) \ } \frac{43}4\sqrt{3}</math>
  
 
[[1999 AHSME Problems/Problem 23|Solution]]
 
[[1999 AHSME Problems/Problem 23|Solution]]
  
 
== Problem 24 ==
 
== Problem 24 ==
 +
Six points on a circle are given. Four of the chords joining pairs of the six points are selected at random. What is the probability that the four chords form a convex quadrilateral?
  
<math> \mathrm{(A) \ } \qquad \mathrm{(B) \ } \qquad \mathrm{(C) \ } \qquad \mathrm{(D) \ } \qquad \mathrm{(E) \ } </math>
+
<math> \mathrm{(A) \ } \frac 1{15} \qquad \mathrm{(B) \ } \frac 1{91} \qquad \mathrm{(C) \ } \frac 1{273} \qquad \mathrm{(D) \ } \frac 1{455} \qquad \mathrm{(E) \ } \frac 1{1365}</math>
  
 
[[1999 AHSME Problems/Problem 24|Solution]]
 
[[1999 AHSME Problems/Problem 24|Solution]]
  
 
== Problem 25 ==
 
== Problem 25 ==
 +
There are unique integers <math>a_{2},a_{3},a_{4},a_{5},a_{6},a_{7}</math> such that
  
<math> \mathrm{(A) \ } \qquad \mathrm{(B) \ } \qquad \mathrm{(C) \ } \qquad \mathrm{(D) \ } \qquad \mathrm{(E) \ } </math>
+
<cmath>\frac {5}{7} = \frac {a_{2}}{2!} + \frac {a_{3}}{3!} + \frac {a_{4}}{4!} + \frac {a_{5}}{5!} + \frac {a_{6}}{6!} + \frac {a_{7}}{7!}</cmath>
 +
 
 +
where <math>0\leq a_{i} < i</math> for <math>i = 2,3,\ldots,7</math>.  Find <math>a_{2} + a_{3} + a_{4} + a_{5} + a_{6} + a_{7}</math>.
 +
 
 +
<math>\textrm{(A)} \ 8 \qquad \textrm{(B)} \ 9 \qquad \textrm{(C)} \ 10 \qquad \textrm{(D)} \ 11 \qquad \textrm{(E)} \ 12</math>
  
 
[[1999 AHSME Problems/Problem 25|Solution]]
 
[[1999 AHSME Problems/Problem 25|Solution]]
  
 
== Problem 26 ==
 
== Problem 26 ==
 +
Three non-overlapping regular plane polygons, at least two of which are congruent, all have sides of length <math>1</math>. The polygons meet at a point <math>A</math> in such a way that the sum of the three interior angles at <math>A</math> is <math>360^{\circ}</math>. Thus the three polygons form a new polygon with <math>A</math> as an interior point. What is the largest possible perimeter that this polygon can have?
  
<math> \mathrm{(A) \ } \qquad \mathrm{(B) \ } \qquad \mathrm{(C) \ } \qquad \mathrm{(D) \ } \qquad \mathrm{(E) \ } </math>
+
<math> \mathrm{(A) \ }12 \qquad \mathrm{(B) \ }14 \qquad \mathrm{(C) \ }18 \qquad \mathrm{(D) \ }21 \qquad \mathrm{(E) \ } 24</math>
  
 
[[1999 AHSME Problems/Problem 26|Solution]]
 
[[1999 AHSME Problems/Problem 26|Solution]]
  
 
== Problem 27 ==
 
== Problem 27 ==
 +
In triangle <math>ABC</math>, <math>3 \sin A + 4 \cos B = 6</math> and <math>4 \sin B + 3 \cos A = 1</math>. Then <math>\angle C</math> in degrees is
  
<math> \mathrm{(A) \ } \qquad \mathrm{(B) \ } \qquad \mathrm{(C) \ } \qquad \mathrm{(D) \ } \qquad \mathrm{(E) \ } </math>
+
<math> \mathrm{(A) \ }30 \qquad \mathrm{(B) \ }60 \qquad \mathrm{(C) \ }90 \qquad \mathrm{(D) \ }120 \qquad \mathrm{(E) \ }150 </math>
  
 
[[1999 AHSME Problems/Problem 27|Solution]]
 
[[1999 AHSME Problems/Problem 27|Solution]]
  
 
== Problem 28 ==
 
== Problem 28 ==
 +
Let <math>x_1, x_2, \ldots , x_n</math> be a sequence of integers such that
 +
 +
<math>\text{(i)}</math> <math>-1 \le x_i \le 2</math> <math>\text{for}</math> <math>i = 1,2, \ldots n</math>
 +
 +
<math>\text{(ii)}</math> <math>x_1 + \cdots + x_n = 19</math>; <math>\text{and}</math>
 +
 +
<math>\text{(iii)}</math> <math>x_1^2 + x_2^2 + \cdots + x_n^2 = 99</math>.
  
<math> \mathrm{(A) \ } \qquad \mathrm{(B) \ } \qquad \mathrm{(C) \ } \qquad \mathrm{(D) \ } \qquad \mathrm{(E) \ } </math>
+
Let <math>m</math> and <math>M</math> be the minimal and maximal possible values of <math>x_1^3 + \cdots + x_n^3</math>, respectively. Then <math>\frac Mm =</math>
 +
 
 +
<math> \mathrm{(A) \ }3 \qquad \mathrm{(B) \ }4 \qquad \mathrm{(C) \ }5 \qquad \mathrm{(D) \ }6 \qquad \mathrm{(E) \ }7 </math>
  
 
[[1999 AHSME Problems/Problem 28|Solution]]
 
[[1999 AHSME Problems/Problem 28|Solution]]
  
 
== Problem 29 ==
 
== Problem 29 ==
 +
A tetrahedron with four equilateral triangular faces has a sphere inscribed within it and a sphere circumscribed about it. For each of the four faces, there is a sphere tangent externally to the face at its center and to the circumscribed sphere. A point <math>P</math> is selected at random inside the circumscribed sphere. The probability that <math>P</math> lies inside one of the five small spheres is closest to
  
<math> \mathrm{(A) \ } \qquad \mathrm{(B) \ } \qquad \mathrm{(C) \ } \qquad \mathrm{(D) \ } \qquad \mathrm{(E) \ } </math>
+
<math> \mathrm{(A) \ }0 \qquad \mathrm{(B) \ }0.1 \qquad \mathrm{(C) \ }0.2 \qquad \mathrm{(D) \ }0.3 \qquad \mathrm{(E) \ }0.4 </math>
  
 
[[1999 AHSME Problems/Problem 29|Solution]]
 
[[1999 AHSME Problems/Problem 29|Solution]]
  
 
== Problem 30 ==
 
== Problem 30 ==
 +
The number of ordered pairs of integers <math>(m,n)</math> for which <math>mn \ge 0</math> and
  
<math> \mathrm{(A) \ } \qquad \mathrm{(B) \ } \qquad \mathrm{(C) \ } \qquad \mathrm{(D) \ } \qquad \mathrm{(E) \ } </math>
+
<cmath>m^3 + n^3 + 99mn = 33^3</cmath>
 +
 
 +
is equal to
 +
 
 +
<math> \mathrm{(A) \ }2 \qquad \mathrm{(B) \ } 3\qquad \mathrm{(C) \ } 33\qquad \mathrm{(D) \ }35 \qquad \mathrm{(E) \ } 99</math>
  
 
[[1999 AHSME Problems/Problem 30|Solution]]
 
[[1999 AHSME Problems/Problem 30|Solution]]
  
  
== See also ==
+
== See Also ==
*[[AHSME]]
+
 
 +
* [[AMC 12 Problems and Solutions]]
 +
* [[Mathematics competition resources]]
 +
 
 +
{{AHSME box|year=1999|before=[[1998 AHSME]]|after=Last AHSME, see [[2000 AMC 12]]}}
 +
 
 +
 
 +
{{MAA Notice}}

Latest revision as of 18:28, 19 June 2023

1999 AHSME (Answer Key)
Printable versions: WikiAoPS ResourcesPDF

Instructions

  1. This is a 30-question, multiple choice test. Each question is followed by answers marked A, B, C, D and E. Only one of these is correct.
  2. You will receive 5 points for each correct answer, 2 points for each problem left unanswered, and 0 points for each incorrect answer.
  3. No aids are permitted other than scratch paper, graph paper, ruler, compass, protractor and erasers.
  4. Figures are not necessarily drawn to scale.
  5. You will have 90 minutes working time to complete the test.
1 2 3 4 5 6 7 8 9 10 11 12 13 14 15 16 17 18 19 20 21 22 23 24 25 26 27 28 29 30

Problem 1

$1 - 2 + 3 -4 + \cdots - 98 + 99 =$

$\mathrm{(A) \ -50 } \qquad \mathrm{(B) \ -49 } \qquad \mathrm{(C) \ 0 } \qquad \mathrm{(D) \ 49 } \qquad \mathrm{(E) \ 50 }$

Solution

Problem 2

Which of the following statements is false?

$\text{(A) All equilateral triangles are congruent to each other.}$

$\text{(B) All equilateral triangles are convex.}$

$\text{(C) All equilateral triangles are equiangular.}$

$\text{(D) All equilateral triangles are regular polygons.}$

$\text{(E) All equilateral triangles are similar to each other.}$

Solution

Problem 3

The number halfway between $1/8$ and $1/10$ is

$\mathrm{(A) \  } \frac 1{80} \qquad \mathrm{(B) \  } \frac 1{40} \qquad \mathrm{(C) \  } \frac 1{18} \qquad \mathrm{(D) \  } \frac 1{9} \qquad \mathrm{(E) \  } \frac 9{80}$

Solution

Problem 4

Find the sum of all prime numbers between $1$ and $100$ that are simultaneously $1$ greater than a multiple of $4$ and $1$ less than a multiple of $5$.

$\mathrm{(A) \ } 118 \qquad \mathrm{(B) \ }137 \qquad \mathrm{(C) \ } 158 \qquad \mathrm{(D) \ } 187 \qquad \mathrm{(E) \ } 245$

Solution

Problem 5

The marked price of a book was $30 \%$ less than the suggested retail price. Alice purchased the book for half the marked price at a Fiftieth Anniversary sale. What percent of the suggested retail price did Alice pay?

$\mathrm{(A) \ }25 \% \qquad \mathrm{(B) \ }30 \% \qquad \mathrm{(C) \ }35 \% \qquad \mathrm{(D) \ }60 \% \qquad \mathrm{(E) \ }65 \%$

Solution

Problem 6

What is the sum of the digits of the decimal form of the product $2^{1999} \cdot 5^{2001}$?

$\mathrm{(A) \ }2 \qquad \mathrm{(B) \ }4 \qquad \mathrm{(C) \ }5 \qquad \mathrm{(D) \ }7 \qquad \mathrm{(E) \ }10$

Solution

Problem 7

What is the largest number of acute angles that a convex hexagon can have?

$\mathrm{(A) \ }2 \qquad \mathrm{(B) \ }3 \qquad \mathrm{(C) \ }4 \qquad \mathrm{(D) \ }5 \qquad \mathrm{(E) \ }6$

Solution

Problem 8

At the end of 1994 Walter was half as old as his grandmother. The sum of the years in which they were born is 3838. How old will Walter be at the end of 1999?

$\mathrm{(A) \ } 48 \qquad \mathrm{(B) \ }49 \qquad \mathrm{(C) \ }53 \qquad \mathrm{(D) \ }55 \qquad \mathrm{(E) \ } 101$

Solution

Problem 9

Before Ashley started a three-hour drive, her car's odometer reading was 29792, a palindrome. (A palindrome is a number that reads the same way from left to right as it does from right to left). At her destination, the odometer reading was another palindrome. If Ashley never exceeded the speed limit of 75 miles per hour, which of the following was her greatest possible average speed?

$\mathrm{(A) \ } 33\frac 13 \qquad \mathrm{(B) \ }53\frac 13 \qquad \mathrm{(C) \ }66\frac 23 \qquad \mathrm{(D) \ }70\frac 13 \qquad \mathrm{(E) \ } 74\frac 13$

Solution

Problem 10

A sealed envelope contains a card with a single digit on it. Three of the following statements are true, and the other is false.

I. The digit is 1.

II. the digit is not 2.

III. The digit is 3.

IV. The digit is not 4.

Which one of the following must necessarily be correct?

$\mathrm{(A) \ I\ is\ true} \qquad \mathrm{(B) \ I\ is\ false} \qquad \mathrm{(C) \ II\ is\ true} \qquad \mathrm{(D) \ III\ is\ true} \qquad \mathrm{(E) \ IV\ is\ false}$

Solution

Problem 11

The student lockers at Olympic High are numbered consecutively beginning with locker number $1$. The plastic digits used to number the lockers cost two cents apiece. Thus, it costs two cents to label locker number $9$ and four cents to label locker number $10$. If it costs $137.94$ to label all the lockers, how many lockers are there at the school?

$\mathrm{(A) \ }2001 \qquad \mathrm{(B) \ }2010 \qquad \mathrm{(C) \ }2100 \qquad \mathrm{(D) \ }2726 \qquad \mathrm{(E) \ }6897$

Solution

Problem 12

What is the maximum number of points of intersection of the graphs of two different fourth degree polynomial functions $y = p(x)$ and $y = q(x)$, each with leading coefficient $1$?

$\textrm{(A)} \ 1 \qquad \textrm{(B)} \ 2 \qquad \textrm{(C)} \ 3 \qquad \textrm{(D)} \ 4 \qquad \textrm{(E)} \ 8$

Solution

Problem 13

Define a sequence of real numbers $a_1, a_2, a_3, \ldots$ by $a_1 = 1$ and $a_{n+1}^3 = 99a_n^3$ for all $n \ge 1$. Then $a_{100}$ equals

$\mathrm{(A) \ } 33^{33} \qquad \mathrm{(B) \ } 33^{99} \qquad \mathrm{(C) \ } 99^{33} \qquad \mathrm{(D) \ }99^{99} \qquad \mathrm{(E) \ none\ of\ the\ above}$

Solution

Problem 14

Four girls - Mary, Aline, Tina, and Hana - sang songs in a concert as trios, with one girl sitting out each time. Hanna sang 7 songs, which was more than any other girl, and Mary sang 4 songs, which was fewer than any other girl. How many songs did these trios sing?

$\mathrm{(A) \ 7 } \qquad \mathrm{(B) \ 8 } \qquad \mathrm{(C) \ 9 } \qquad \mathrm{(D) \ 10 } \qquad \mathrm{(E) \ 11 }$

Solution

Problem 15

Let $x$ be a real number such that $\sec x - \tan x = 2$. Then $\sec x + \tan x =$

$\mathrm{(A) \ } 0.1 \qquad \mathrm{(B) \ } 0.2 \qquad \mathrm{(C) \ } 0.3 \qquad \mathrm{(D) \ } 0.4 \qquad \mathrm{(E) \ } 0.5$

Solution

Problem 16

What is the radius of a circle inscribed in a rhombus with diagonals of length $10$ and $24$?

$\mathrm{(A) \ }4 \qquad \mathrm{(B) \ }\frac {58}{13} \qquad \mathrm{(C) \ }\frac{60}{13} \qquad \mathrm{(D) \ }5 \qquad \mathrm{(E) \ }6$

Solution

Problem 17

Let $P(x)$ be a polynomial such that when $P(x)$ is divided by $x-19$, the remainder is $99$, and when $P(x)$ is divided by $x - 99$, the remainder is $19$. What is the remainder when $P(x)$ is divided by $(x-19)(x-99)$?

$\mathrm{(A) \ } -x + 80 \qquad \mathrm{(B) \ } x + 80 \qquad \mathrm{(C) \ } -x + 118 \qquad \mathrm{(D) \ } x + 118 \qquad \mathrm{(E) \ } 0$

Solution

Problem 18

How many zeros does $f(x) = \cos(\log x)$ have on the interval $0 < x < 1$?

$\mathrm{(A) \ } 0 \qquad \mathrm{(B) \ } 1 \qquad \mathrm{(C) \ } 2 \qquad \mathrm{(D) \ } 10 \qquad \mathrm{(E) \ } \text{infinitely\ many}$

Solution

Problem 19

Consider all triangles $ABC$ satisfying in the following conditions: $AB = AC$, $D$ is a point on $\overline{AC}$ for which $\overline{BD} \perp \overline{AC}$, $AC$ and $CD$ are integers, and $BD^{2} = 57$. Among all such triangles, the smallest possible value of $AC$ is

[asy] pair A,B,C,D;  A=(5,12); B=origin; C=(10,0); D=(8.52071005917,3.55029585799); draw(A--B--C--cycle); draw(B--D); label("$A$",A,N); label("$B$",B,SW); label("$C$",C,SE); label("$D$",D,NE); [/asy]

$\textrm{(A)} \ 9 \qquad \textrm{(B)} \ 10 \qquad \textrm{(C)} \ 11 \qquad \textrm{(D)} \ 12 \qquad \textrm{(E)} \ 13$

Solution

Problem 20

The sequence $a_{1},a_{2},a_{3},\ldots$ satisfies $a_{1} = 19,a_{9} = 99$, and, for all $n\geq 3$, $a_{n}$ is the arithmetic mean of the first $n - 1$ terms. Find $a_2$.

$\textrm{(A)} \ 29 \qquad \textrm{(B)} \ 59 \qquad \textrm{(C)} \ 79 \qquad \textrm{(D)} \ 99 \qquad \textrm{(E)} \ 179$

Solution

Problem 21

A circle is circumscribed about a triangle with sides $20,21,$ and $29,$ thus dividing the interior of the circle into four regions. Let $A,B,$ and $C$ be the areas of the non-triangular regions, with $C$ be the largest. Then

$\mathrm{(A) \ }A+B=C \qquad \mathrm{(B) \ }A+B+210=C \qquad \mathrm{(C) \ }A^2+B^2=C^2 \qquad \mathrm{(D) \ }20A+21B=29C \qquad \mathrm{(E) \ } \frac 1{A^2}+\frac 1{B^2}= \frac 1{C^2}$

Solution

Problem 22

The graphs of $y = -|x-a| + b$ and $y = |x-c| + d$ intersect at points $(2,5)$ and $(8,3)$. Find $a+c$.

$\mathrm{(A) \ } 7 \qquad \mathrm{(B) \ } 8 \qquad \mathrm{(C) \ } 10 \qquad \mathrm{(D) \ } 13\qquad \mathrm{(E) \ } 18$

Solution

Problem 23

The equiangular convex hexagon $ABCDEF$ has $AB = 1, BC = 4, CD = 2,$ and $DE = 4.$ The area of the hexagon is

$\mathrm{(A) \ } \frac {15}2\sqrt{3} \qquad \mathrm{(B) \ }9\sqrt{3} \qquad \mathrm{(C) \ }16 \qquad \mathrm{(D) \ }\frac{39}4\sqrt{3} \qquad \mathrm{(E) \ } \frac{43}4\sqrt{3}$

Solution

Problem 24

Six points on a circle are given. Four of the chords joining pairs of the six points are selected at random. What is the probability that the four chords form a convex quadrilateral?

$\mathrm{(A) \ } \frac 1{15} \qquad \mathrm{(B) \ } \frac 1{91} \qquad \mathrm{(C) \ } \frac 1{273} \qquad \mathrm{(D) \ } \frac 1{455} \qquad \mathrm{(E) \ } \frac 1{1365}$

Solution

Problem 25

There are unique integers $a_{2},a_{3},a_{4},a_{5},a_{6},a_{7}$ such that

\[\frac {5}{7} = \frac {a_{2}}{2!} + \frac {a_{3}}{3!} + \frac {a_{4}}{4!} + \frac {a_{5}}{5!} + \frac {a_{6}}{6!} + \frac {a_{7}}{7!}\]

where $0\leq a_{i} < i$ for $i = 2,3,\ldots,7$. Find $a_{2} + a_{3} + a_{4} + a_{5} + a_{6} + a_{7}$.

$\textrm{(A)} \ 8 \qquad \textrm{(B)} \ 9 \qquad \textrm{(C)} \ 10 \qquad \textrm{(D)} \ 11 \qquad \textrm{(E)} \ 12$

Solution

Problem 26

Three non-overlapping regular plane polygons, at least two of which are congruent, all have sides of length $1$. The polygons meet at a point $A$ in such a way that the sum of the three interior angles at $A$ is $360^{\circ}$. Thus the three polygons form a new polygon with $A$ as an interior point. What is the largest possible perimeter that this polygon can have?

$\mathrm{(A) \ }12 \qquad \mathrm{(B) \ }14 \qquad \mathrm{(C) \ }18 \qquad \mathrm{(D) \ }21 \qquad \mathrm{(E) \ } 24$

Solution

Problem 27

In triangle $ABC$, $3 \sin A + 4 \cos B = 6$ and $4 \sin B + 3 \cos A = 1$. Then $\angle C$ in degrees is

$\mathrm{(A) \ }30 \qquad \mathrm{(B) \ }60 \qquad \mathrm{(C) \ }90 \qquad \mathrm{(D) \ }120 \qquad \mathrm{(E) \ }150$

Solution

Problem 28

Let $x_1, x_2, \ldots , x_n$ be a sequence of integers such that

$\text{(i)}$ $-1 \le x_i \le 2$ $\text{for}$ $i = 1,2, \ldots n$

$\text{(ii)}$ $x_1 + \cdots + x_n = 19$; $\text{and}$

$\text{(iii)}$ $x_1^2 + x_2^2 + \cdots + x_n^2 = 99$.

Let $m$ and $M$ be the minimal and maximal possible values of $x_1^3 + \cdots + x_n^3$, respectively. Then $\frac Mm =$

$\mathrm{(A) \ }3 \qquad \mathrm{(B) \ }4 \qquad \mathrm{(C) \ }5 \qquad \mathrm{(D) \ }6 \qquad \mathrm{(E) \ }7$

Solution

Problem 29

A tetrahedron with four equilateral triangular faces has a sphere inscribed within it and a sphere circumscribed about it. For each of the four faces, there is a sphere tangent externally to the face at its center and to the circumscribed sphere. A point $P$ is selected at random inside the circumscribed sphere. The probability that $P$ lies inside one of the five small spheres is closest to

$\mathrm{(A) \ }0 \qquad \mathrm{(B) \ }0.1 \qquad \mathrm{(C) \ }0.2 \qquad \mathrm{(D) \ }0.3 \qquad \mathrm{(E) \ }0.4$

Solution

Problem 30

The number of ordered pairs of integers $(m,n)$ for which $mn \ge 0$ and

\[m^3 + n^3 + 99mn = 33^3\]

is equal to

$\mathrm{(A) \ }2 \qquad \mathrm{(B) \ } 3\qquad \mathrm{(C) \ } 33\qquad \mathrm{(D) \ }35 \qquad \mathrm{(E) \ } 99$

Solution


See Also

1999 AHSME (ProblemsAnswer KeyResources)
Preceded by
1998 AHSME
Followed by
Last AHSME, see 2000 AMC 12
1 2 3 4 5 6 7 8 9 10 11 12 13 14 15 16 17 18 19 20 21 22 23 24 25 26 27 28 29 30
All AHSME Problems and Solutions


The problems on this page are copyrighted by the Mathematical Association of America's American Mathematics Competitions. AMC logo.png